1answer.
Ask question
Login Signup
Ask question
All categories
  • English
  • Mathematics
  • Social Studies
  • Business
  • History
  • Health
  • Geography
  • Biology
  • Physics
  • Chemistry
  • Computers and Technology
  • Arts
  • World Languages
  • Spanish
  • French
  • German
  • Advanced Placement (AP)
  • SAT
  • Medicine
  • Law
  • Engineering
notsponge [240]
3 years ago
5

What is the value of (10 – 6)2 + 6 • 2 – (3 + 2)2? A. 10 B. 3 C. –1 D. –5

Mathematics
2 answers:
Ira Lisetskai [31]3 years ago
8 0
I believe the answer is A,
(10-6)2+6•2-(3+2)2
4•2+6•2-5•2
8+12-10
20-10
10
To solve these questions use PEMDAS
First you do the Parentheses
Then solve Exponents
After that Multiple and divide left to right
Lastly Add and subtract left to right
IrinaVladis [17]3 years ago
4 0

Answer:

A.  10

Step-by-step explanation:

(10-6)2 + 6*2 - (3+2)2

20-12+12-6+4

8+12-10

20-10

10

You might be interested in
Find the slope of the equation: y = 2x + 5
irga5000 [103]

Answer:

The slope is 2

Step-by-step explanation:

8 0
3 years ago
You operate a gaming website, www.mudbeast.net, where users must pay a small fee to log on. When you charged $4 the demand was 5
sertanlavr [38]
A) Demand function

price (x)          demand (D(x))

4                      540

3.50                  810

D - 540          810 - 540
----------- =  -----------------
x - 4               3.50 - 4

D - 540
----------- = - 540
x - 4

D - 540 = - 540(x - 4)

D = -540x + 2160 + 540

D = 2700 - 540x

D(x) = 2700 - 540x

Revenue function, R(x)

R(x) = price * demand = x * D(x)

R(x) = x* (2700 - 540x) = 2700x - 540x^2

b) Profit, P(x)

profit = revenue - cost

P(x) = R(x) - 30

P(x) = [2700x - 540x^2] - 30

P(x) = 2700x - 540x^2 - 30

Largest possible profit => vertex of the parabola

vertex of 2700x - 540x^2 - 30

When you calculate the vertex you find x = 5 /2

=> P(x) = 3345

Answer: you should charge a log-on fee of $2.5 to have the largest profit, which is $3345.




5 0
3 years ago
Someone please help me!!!!!
Katarina [22]

1. Answer (D). By the law of sines, we have \frac{a}{\sin A}=\frac{b}{\sin B}=\frac{c}{\sin C} in any \triangle ABC.

2. Answer (C). The law of cosines, c^2=a^2+b^2-2ab\cos C,accepts up to three sides and an angle as an input.

3. Answer (D). Although this triangle is right, we are not given enough information to uniquely determine its sides and angles - here, we need either one more side or one more angle.

4. Answer (D). Don't get tripped up by answer choice (C) - this is just a rearrangement of the statement of the law of cosines. In choice (D), the signs of a^2 and 2ab\cos C are reversed.

5. Answer (B). By the law of sines, we have \frac{5}{\sin 40^\circ}=\frac{3}{\sin\theta}. Solving gives \theta\approx \boxed{23^\circ},157^\circ. Note that this is the <em>ambiguous (SSA) case</em> of the law of sines, where the given measures could specify one triangle, two triangles, or none at all!

6. Answer (A). Since we know all three sides and none of the angles, starting with the law of sines will not help, so we begin with the law of cosines to find one angle; from there, we can use the law of sines to find the remaining angles.

6 0
3 years ago
Mark is playing soccer. He is
eimsori [14]

Answer:

it would be 3 yards because it went 2° to the right

7 0
3 years ago
I really need help..!
Doss [256]

Answer:

B: 1/6

Step-by-step explanation:

make 1/4 to 3/12. 7/12 + 3/12 is 10/12. So the green is 2/12. 2/12 simplified is 1/6.

4 0
3 years ago
Read 2 more answers
Other questions:
  • Is 10 times as much as 0.24?
    13·1 answer
  • A watercolor painting is 16 inches long by 12 inches wide. Ramon makes a border around the watercolor
    14·1 answer
  • What is 1/4as a decimal
    13·2 answers
  • Gerald is constructing a line parallel to line l through point P. He begins by drawing line m through points P and Q. He then dr
    5·2 answers
  • For each of the following pairs of numbers, find the gcd of the two numbers, and express the gcd as a linear combination of the
    12·1 answer
  • How do you eliminate the fractions on this to solve by elimination ????????
    15·2 answers
  • Please answer correctly !!!!!!!!!! Will mark Brianliest !!!!!!!!!!!!!!!!
    15·1 answer
  • Find the coordinate of X if XY=1 and the coordinate of Y is 0.
    6·1 answer
  • Help w y math shawtys
    15·1 answer
  • A translation moves P(0,5) to P'(3,1). What are the coordinates of the image
    9·1 answer
Add answer
Login
Not registered? Fast signup
Signup
Login Signup
Ask question!